Last visit was: 26 Apr 2024, 00:42 It is currently 26 Apr 2024, 00:42

Close
GMAT Club Daily Prep
Thank you for using the timer - this advanced tool can estimate your performance and suggest more practice questions. We have subscribed you to Daily Prep Questions via email.

Customized
for You

we will pick new questions that match your level based on your Timer History

Track
Your Progress

every week, we’ll send you an estimated GMAT score based on your performance

Practice
Pays

we will pick new questions that match your level based on your Timer History
Not interested in getting valuable practice questions and articles delivered to your email? No problem, unsubscribe here.
Close
Request Expert Reply
Confirm Cancel
SORT BY:
Date
Tags:
Show Tags
Hide Tags
GMAT Tutor
Joined: 24 Jun 2008
Posts: 4128
Own Kudos [?]: 9245 [3]
Given Kudos: 91
 Q51  V47
Send PM
User avatar
Manager
Manager
Joined: 17 Nov 2011
Status:Employed
Posts: 67
Own Kudos [?]: 433 [4]
Given Kudos: 10
Location: Pakistan
Concentration: International Business, Marketing
GMAT 1: 720 Q49 V40
GPA: 3.2
WE:Business Development (Internet and New Media)
Send PM
User avatar
Manager
Manager
Joined: 17 Nov 2011
Status:Employed
Posts: 67
Own Kudos [?]: 433 [15]
Given Kudos: 10
Location: Pakistan
Concentration: International Business, Marketing
GMAT 1: 720 Q49 V40
GPA: 3.2
WE:Business Development (Internet and New Media)
Send PM
User avatar
Manager
Manager
Joined: 17 Nov 2011
Status:Employed
Posts: 67
Own Kudos [?]: 433 [19]
Given Kudos: 10
Location: Pakistan
Concentration: International Business, Marketing
GMAT 1: 720 Q49 V40
GPA: 3.2
WE:Business Development (Internet and New Media)
Send PM
Re: NEW!!! Tough and tricky exponents and roots questions [#permalink]
6
Kudos
13
Bookmarks
Q4. What is the value of \(5+4*5+4*5^2+4*5^3+4*5^4+4*5^5\)

This one was actually the simplest I thought. Here is how:

\(5+4*5+4*5^2+4*5^3+4*5^4+4*5^5\)

So \(5*(1+4)+4*5^2+4*5^3+4*5^4+4*5^5\)
So \(5^2+4*5^2+4*5^3+4*5^4+4*5^5\)
So \(5^2*(1+4)+4*5^3+4*5^4+4*5^5\)
So \(5^3+4*5^3+4*5^4+4*5^5\)
So \(5^3*(1+4)+4*5^4+4*5^5\)

So every expression behind contributes a power of 1 to the one in front of it. We just need to see the last which is \(5^5\)
Keep solving and you come to a total of \(5^6\)

Hence Answer = \(5^6\)
User avatar
Manager
Manager
Joined: 06 Oct 2011
Posts: 157
Own Kudos [?]: 83 [14]
Given Kudos: 10
GMAT Date: 06-30-2012
GPA: 3.7
WE:Accounting (Insurance)
Send PM
Re: NEW!!! Tough and tricky exponents and roots questions [#permalink]
13
Kudos
“The only true wisdom is in knowing you know nothing.”
― Socrates


back to studying!
User avatar
Manager
Manager
Joined: 13 Jun 2011
Status:Do till 740 :)
Posts: 61
Own Kudos [?]: 31 [2]
Given Kudos: 19
Concentration: Strategy, General Management
GMAT 1: 460 Q35 V20
GPA: 3.6
WE:Consulting (Computer Software)
Send PM
Re: NEW!!! Tough and tricky exponents and roots questions [#permalink]
2
Bookmarks
Quote:
What is the units digit of (17^3)^4-1973^{3^2}?
A. 0
B. 2
C. 4
D. 6
E. 8




Quote:
So, we have that the units digit of (17^3)^4=17^{12} is 1 and the units digit of 1973^3^2=1973^9 is 3. Also note that the second number is mush larger then the first one, thus their difference will be negative, something like 11-13=-2, which gives the final answer that the units digit of (17^3)^4-1973^{3^2} is 2.

Answer B.



I agree with all the steps above but i have this doubt in this

unit digit of 1st number is 1.
Unit digit of second number is 3

1-3
would it not mean
unit digit is 8?
for eg if we have
551 -853 ..Unit digit will be 2
but what if it is 551-453 .unit digit will be 8?

so how do we know which is the larger number of these, although it says X-Y?
Please let me know,.
Math Expert
Joined: 02 Sep 2009
Posts: 92919
Own Kudos [?]: 619084 [4]
Given Kudos: 81595
Send PM
Re: NEW!!! Tough and tricky exponents and roots questions [#permalink]
3
Kudos
1
Bookmarks
Expert Reply
shankar245 wrote:
Quote:
What is the units digit of (17^3)^4-1973^{3^2}?
A. 0
B. 2
C. 4
D. 6
E. 8




Quote:
So, we have that the units digit of (17^3)^4=17^{12} is 1 and the units digit of 1973^3^2=1973^9 is 3. Also notice that the second number is much larger then the first one, thus their difference will be negative, something like 11-13=-2, which gives the final answer that the units digit of (17^3)^4-1973^{3^2} is 2.

Answer B.



I agree with all the steps above but i have this doubt in this

unit digit of 1st number is 1.
Unit digit of second number is 3

1-3
would it not mean
unit digit is 8?
for eg if we have
551 -853 ..Unit digit will be 2
but what if it is 551-453 .unit digit will be 8?

so how do we know which is the larger number of these, although it says X-Y?
Please let me know,.


That's a little trap there.

Notice that the second number is much larger then the first one, thus their difference will be negative, something like 11-13=-2, which gives the final answer that the units digit of \((17^3)^4-1973^{3^2}\) is 2.

So, as you can see the cases like 551-453 (larger number minus smaller number) are not possible.

Why is second number much larger then the first one? Consider this, even if we had \((100^3)^4\) (instead of \((17^3)^4\)) and \(1000^{(3^2)}\) (instead of \(1973^{(3^2)}\)) --> \((100^3)^4=100^{12}=10^{24}\) and \(1000^{(3^2)}=1,000^9=10^{27}\).

Hope it's clear.
User avatar
Intern
Intern
Joined: 03 Nov 2010
Posts: 9
Own Kudos [?]: 3 [0]
Given Kudos: 0
Send PM
Re: NEW!!! Tough and tricky exponents and roots questions [#permalink]
Bunuel wrote:
4^{\sqrt{y}


For Question #3.
I'm confused why 4 squareroot Y = (2 squareroot Y)^2 instead of (2 ^2 squareroot Y).

Not sure I understand the properties correctly...
Math Expert
Joined: 02 Sep 2009
Posts: 92919
Own Kudos [?]: 619084 [0]
Given Kudos: 81595
Send PM
Re: NEW!!! Tough and tricky exponents and roots questions [#permalink]
Expert Reply
KGG88 wrote:
Bunuel wrote:
4^{\sqrt{y}


For Question #3.
I'm confused why 4 squareroot Y = (2 squareroot Y)^2 instead of (2 ^2 squareroot Y).

Not sure I understand the properties correctly...



\(4^{\sqrt{y}}=(2^2)^{\sqrt{y}}=2^{2*\sqrt{y}}=(2^{\sqrt{y}})^2\).

Check this for properties: tough-and-tricky-exponents-and-roots-questions-125956.html#p1027888

Hope it helps.
Math Expert
Joined: 02 Sep 2009
Posts: 92919
Own Kudos [?]: 619084 [6]
Given Kudos: 81595
Send PM
Re: NEW!!! Tough and tricky exponents and roots questions [#permalink]
2
Kudos
4
Bookmarks
Expert Reply
kuttingchai wrote:
Bunuel wrote:
2. What is the units digit of \((17^3)^4-1973^{3^2}\)?
A. 0
B. 2
C. 4
D. 6
E. 8

Must know for the GMAT:
I. The units digit of \((abc)^n\) is the same as that of \(c^n\), which means that the units digit of \((17^3)^4\) is that same as that of \((7^3)^4\) and the units digit of \(1973^{3^2}\) is that same as that of \(3^{3^2}\).

II. If exponentiation is indicated by stacked symbols, the rule is to work from the top down, thus:
\(a^m^n=a^{(m^n)}\) and not \((a^m)^n\), which on the other hand equals to \(a^{mn}\).

So:
\((a^m)^n=a^{mn}\);

\(a^m^n=a^{(m^n)}\).

Thus, \((7^3)^4=7^{(3*4)}=7^{12}\) and \(3^{3^2}=3^{(3^2)}=3^9\).

III. The units digit of integers in positive integer power repeats in specific pattern (cyclicity): The units digit of 7 and 3 in positive integer power repeats in patterns of 4:

1. 7^1=7 (last digit is 7)
2. 7^2=9 (last digit is 9)
3. 7^3=3 (last digit is 3)
4. 7^4=1 (last digit is 1)

5. 7^5=7 (last digit is 7 again!)
...

1. 3^1=3 (last digit is 3)
2. 3^2=9 (last digit is 9)
3. 3^3=27 (last digit is 7)
4. 3^4=81 (last digit is 1)

5. 3^5=243 (last digit is 3 again!)
...

Thus th units digit of \(7^{12}\) will be 1 (4th in pattern, as 12 is a multiple of cyclicty number 4) and the units digit of \(3^9\) will be 3 (first in pattern, as 9=4*2+1).

So, we have that the units digit of \((17^3)^4=17^{12}\) is 1 and the units digit of \(1973^3^2=1973^9\) is 3. Also notice that the second number is much larger then the first one, thus their difference will be negative, something like 11-13=-2, which gives the final answer that the units digit of \((17^3)^4-1973^{3^2}\) is 2.

Answer B.



Hey Bunuel,

I understood the logic behind finding the unit places, but how can u determine if the reminder is 2 or 8

in this example
(17^12) - (1973^9) = unit place is 2 (agreed- i actually did the calculation using calculator ;) )
because : (17^12) unit place = 1
(1973^9) unit place = 3

but if we have just - (7^12) - (3^9) or (3^9) - (7^12) we have unit place as "8"

3^9 = 19683 - unit place will still be 2
7^12 = 13841287201 - unit place will still be 1

so my question is how will u determine if the answer is 2/8? clearly we have 2 different answers??
am i missing anything??


The units digit of \(17^{12}-1973^{9}\) is 2 and not 8 since \(1973^{9}\) is much larger number than \(17^{12}\), thus their difference will be negative, something like 11-13=-2.

If we had something like 21-13 (if the first number were greater than the second one), then the units digit of their difference would be 8.

Hope i's clear.
User avatar
Director
Director
Joined: 22 Mar 2011
Posts: 520
Own Kudos [?]: 2136 [4]
Given Kudos: 43
WE:Science (Education)
Send PM
Re: NEW!!! Tough and tricky exponents and roots questions [#permalink]
3
Kudos
1
Bookmarks
Bunuel wrote:
SOLUTIONS:

1. What is the value of \(\sqrt{25+10\sqrt{6}}+\sqrt{25-10\sqrt{6}}\)?
A. \(2\sqrt{5}\)
B. \(\sqrt{55}\)
C. \(2\sqrt{15}\)
D. 50
E. 60

Square the given expression to get rid of the roots, though don't forget to un-square the value you get at the end to balance this operation and obtain the right answer:

Must know fro the GMAT: \((x+y)^2=x^2+2xy+y^2\) (while \((x-y)^2=x^2-2xy+y^2\)).

So we get: \((\sqrt{25+10\sqrt{6}}+\sqrt{25-10\sqrt{6}})^2=(\sqrt{25+10\sqrt{6}})^2+2(\sqrt{25+10\sqrt{6}})(\sqrt{25-10\sqrt{6}})+(\sqrt{25-10\sqrt{6}})^2=\)
\(=(25+10\sqrt{6})+2(\sqrt{25+10\sqrt{6}})(\sqrt{25-10\sqrt{6}})+(25-10\sqrt{6})\).

Note that sum of the first and the third terms simplifies to \((25+10\sqrt{6})+(25-10\sqrt{6})=50\), so we have \(50+2(\sqrt{25+10\sqrt{6}})(\sqrt{25-10\sqrt{6}})\) --> \(50+2(\sqrt{25+10\sqrt{6}})(\sqrt{25-10\sqrt{6}})=50+2\sqrt{(25+10\sqrt{6})(25-10\sqrt{6})}\).

Also must know for the GMAT: \((x+y)(x-y)=x^2-y^2\), thus \(50+2\sqrt{(25+10\sqrt{6})(25-10\sqrt{6})}=50+2\sqrt{25^2-(10\sqrt{6})^2)}=50+2\sqrt{625-600}=50+2\sqrt{25}=60\).

Recall that we should un-square this value to get the right the answer: \(\sqrt{60}=2\sqrt{15}\).

Answer: C.


Another way to do it, using the same formulas:

\(\sqrt{25+10\sqrt{6}}+\sqrt{25-10\sqrt{6}}=\sqrt{5(5+2\sqrt{6})}+\sqrt{5(5-2\sqrt{6})}=\sqrt{5}\sqrt{5+2\sqrt{6}}+\sqrt{5}\sqrt{5-2\sqrt{6}}=\)
\(=\sqrt{5}(\sqrt{5+2\sqrt{6}}+\sqrt{5-2\sqrt{6}})=\sqrt{5}(\sqrt{(\sqrt{3}+\sqrt{2})^2}+\sqrt{(\sqrt{3}-\sqrt{2})^2})=\sqrt{5}(\sqrt{3}+\sqrt{2}+\sqrt{3}-\sqrt{2})=\sqrt{5}(2\sqrt{3})=2\sqrt{15}\)

Originally posted by EvaJager on 29 Jun 2012, 11:34.
Last edited by EvaJager on 15 Oct 2012, 01:35, edited 1 time in total.
User avatar
Manager
Manager
Joined: 22 Dec 2011
Posts: 175
Own Kudos [?]: 1043 [0]
Given Kudos: 32
Send PM
Re: NEW!!! Tough and tricky exponents and roots questions [#permalink]
Bunuel wrote:
8. If \(x\) is a positive number and equals to \(\sqrt{6+{\sqrt{6+\sqrt{6+\sqrt{6+...}}}}}\), where the given expression extends to an infinite number of roots, then what is the value of x?
A. \(\sqrt{6}\)
B. 3
C. \(1+\sqrt{6}\)
D. \(2\sqrt{3}\)
E. 6

Given: \(x>0\) and \(x=\sqrt{6+{\sqrt{6+\sqrt{6+\sqrt{6+...}}}}}\) --> \(x=\sqrt{6+({\sqrt{6+\sqrt{6+\sqrt{6+...})}}}}\), as the expression under the square root extends infinitely then expression in brackets would equal to \(x\) itself and we can safely replace it with \(x\) and rewrite the given expression as \(x=\sqrt{6+x}\). Square both sides: \(x^2=6+x\) --> \((x+2)(x-3)=0\) --> \(x=-2\) or \(x=3\), but since \(x>0\) then: \(x=3\).

Answer: B.


Hi Bunuel - All sols are crystal clear except this one logic mentioned here. I'm not able to understand the below part. Could you please elaborate?

\(x=\sqrt{6+({\sqrt{6+\sqrt{6+\sqrt{6+...})}}}}\), as the expression under the square root extends infinitely then expression in brackets would equal to \(x\) itself and we can safely replace it with \(x\) and rewrite the given expression as \(x=\sqrt{6+x}\).

Cheers
Math Expert
Joined: 02 Sep 2009
Posts: 92919
Own Kudos [?]: 619084 [1]
Given Kudos: 81595
Send PM
Re: NEW!!! Tough and tricky exponents and roots questions [#permalink]
1
Kudos
Expert Reply
Jp27 wrote:
Bunuel wrote:
8. If \(x\) is a positive number and equals to \(\sqrt{6+{\sqrt{6+\sqrt{6+\sqrt{6+...}}}}}\), where the given expression extends to an infinite number of roots, then what is the value of x?
A. \(\sqrt{6}\)
B. 3
C. \(1+\sqrt{6}\)
D. \(2\sqrt{3}\)
E. 6

Given: \(x>0\) and \(x=\sqrt{6+{\sqrt{6+\sqrt{6+\sqrt{6+...}}}}}\) --> \(x=\sqrt{6+({\sqrt{6+\sqrt{6+\sqrt{6+...})}}}}\), as the expression under the square root extends infinitely then expression in brackets would equal to \(x\) itself and we can safely replace it with \(x\) and rewrite the given expression as \(x=\sqrt{6+x}\). Square both sides: \(x^2=6+x\) --> \((x+2)(x-3)=0\) --> \(x=-2\) or \(x=3\), but since \(x>0\) then: \(x=3\).

Answer: B.


Hi Bunuel - All sols are crystal clear except this one logic mentioned here. I'm not able to understand the below part. Could you please elaborate?

\(x=\sqrt{6+({\sqrt{6+\sqrt{6+\sqrt{6+...})}}}}\), as the expression under the square root extends infinitely then expression in brackets would equal to \(x\) itself and we can safely replace it with \(x\) and rewrite the given expression as \(x=\sqrt{6+x}\).

Cheers


Given: \(x=\sqrt{6+({\sqrt{6+\sqrt{6+\sqrt{6+...})}}}}\). Consier the expression in brackets: \((\sqrt{6+\sqrt{6+\sqrt{6+...}}})\). It's the same as the right hand side of the initial expression, thus it also equals to \(x\). When replaced we'll have: \(x=\sqrt{6+x}\).

Hope it's clear.
User avatar
Intern
Intern
Joined: 13 Jun 2010
Posts: 11
Own Kudos [?]: 186 [0]
Given Kudos: 0
Send PM
Re: NEW!!! Tough and tricky exponents and roots questions [#permalink]
Thanks Bunuel for these interesting problems.

With respect to problem number 2

"So, we have that the units digit of is 1 and the units digit of is 3. Also notice that the second number is much larger then the first one, thus their difference will be negative, something like 11-13=-2, which gives the final answer that the units digit of is 2."

I don't understand the above statement. If the last digit is 1 for the smaller number and the last digit is 3 for the larger number.
And we are trying to find different between smaller number - larger number

eg 757571 - 58299374483. We borrow one from the previous digit and make 1 as 11 and then subtract 3 from 11. So shouldn't the last digit be 8.

I am unable to think any other way. Please let me know where I am going wrong.

Thanks
gmatrant
Math Expert
Joined: 02 Sep 2009
Posts: 92919
Own Kudos [?]: 619084 [0]
Given Kudos: 81595
Send PM
Re: NEW!!! Tough and tricky exponents and roots questions [#permalink]
Expert Reply
gmatrant wrote:
Thanks Bunuel for these interesting problems.

With respect to problem number 2

"So, we have that the units digit of is 1 and the units digit of is 3. Also notice that the second number is much larger then the first one, thus their difference will be negative, something like 11-13=-2, which gives the final answer that the units digit of is 2."

I don't understand the above statement. If the last digit is 1 for the smaller number and the last digit is 3 for the larger number.
And we are trying to find different between smaller number - larger number

eg 757571 - 58299374483. We borrow one from the previous digit and make 1 as 11 and then subtract 3 from 11. So shouldn't the last digit be 8.

I am unable to think any other way. Please let me know where I am going wrong.

Thanks
gmatrant


First of all 757,571 - 58,299,374,483 = -58,298,616,912

You could consider easier cases: 11-13=-2, 11-23=-12, 21-83=-62, ...
User avatar
Intern
Intern
Joined: 03 Sep 2011
Posts: 13
Own Kudos [?]: 7 [3]
Given Kudos: 4
Send PM
Re: NEW!!! Tough and tricky exponents and roots questions [#permalink]
3
Bookmarks
You talk about even and odd in this problem. Wouldn't this not be enough? Since 26 has factors of 2 and 13, even if you had added some even numbers in the product it would still not have sufficed? You would need a 13, 39 etc ? Is my reasoning correct?

Bunuel wrote:
5. If \(x=23^2*25^4*27^6*29^8\) and is a multiple of \(26^n\), where \(n\) is a non-negative integer, then what is the value of \(n^{26}-26^n\)?
A. -26
B. -25
C. -1
D. 0
E. 1

\(23^2*25^4*27^6*29^8=odd*odd*odd*odd=odd\) so \(x\) is an odd number. The only way it to be a multiple of \(26^n\) (even number in integer power) is when \(n=0\), in this case \(26^n=26^0=1\) and 1 is a factor of every integer. Thus \(n=0\) --> \(n^{26}-26^n=0^{26}-26^0=0-1=-1\). Must know for the GMAT: \(a^0=1\), for \(a\neq{0}\) - any nonzero number to the power of 0 is 1. Important note: the case of 0^0 is not tested on the GMAT.

Answer: C.
Math Expert
Joined: 02 Sep 2009
Posts: 92919
Own Kudos [?]: 619084 [2]
Given Kudos: 81595
Send PM
Re: NEW!!! Tough and tricky exponents and roots questions [#permalink]
1
Kudos
1
Bookmarks
Expert Reply
jeremystaub28 wrote:
You talk about even and odd in this problem. Wouldn't this not be enough? Since 26 has factors of 2 and 13, even if you had added some even numbers in the product it would still not have sufficed? You would need a 13, 39 etc ? Is my reasoning correct?

Bunuel wrote:
5. If \(x=23^2*25^4*27^6*29^8\) and is a multiple of \(26^n\), where \(n\) is a non-negative integer, then what is the value of \(n^{26}-26^n\)?
A. -26
B. -25
C. -1
D. 0
E. 1

\(23^2*25^4*27^6*29^8=odd*odd*odd*odd=odd\) so \(x\) is an odd number. The only way it to be a multiple of \(26^n\) (even number in integer power) is when \(n=0\), in this case \(26^n=26^0=1\) and 1 is a factor of every integer. Thus \(n=0\) --> \(n^{26}-26^n=0^{26}-26^0=0-1=-1\). Must know for the GMAT: \(a^0=1\), for \(a\neq{0}\) - any nonzero number to the power of 0 is 1. Important note: the case of 0^0 is not tested on the GMAT.

Answer: C.


The fact that the product is odd, is already enough to say that it cannot be a multiple of 26^n unless n=0.
Manager
Manager
Joined: 16 Jan 2013
Posts: 66
Own Kudos [?]: 21 [0]
Given Kudos: 1323
Location: Bangladesh
GMAT 1: 490 Q41 V18
GMAT 2: 610 Q45 V28
GPA: 2.75
Send PM
Re: NEW!!! Tough and tricky exponents and roots questions [#permalink]
Note that we need approximate value of the given expression. Now, \(22^{22x}\) is much larger number than \(22^{2x}\). Hence \(22^{22x}-22^{2x}\) will be very close to \(22^{22x}\) itself, basically \(22^{2x}\) is negligible in this case. The same way \(11^{11x}-11^x\) will be very close to \(11^{11x}\) itself.


Hello Bunuel,
Nice explanations. I am a little confused with the fact which you mentioned as "negligible" in this math. Wouldn't this deduction change the result of the answer?

Thanks. :)
Math Expert
Joined: 02 Sep 2009
Posts: 92919
Own Kudos [?]: 619084 [2]
Given Kudos: 81595
Send PM
Re: NEW!!! Tough and tricky exponents and roots questions [#permalink]
1
Kudos
1
Bookmarks
Expert Reply
ranaazad wrote:
Note that we need approximate value of the given expression. Now, \(22^{22x}\) is much larger number than \(22^{2x}\). Hence \(22^{22x}-22^{2x}\) will be very close to \(22^{22x}\) itself, basically \(22^{2x}\) is negligible in this case. The same way \(11^{11x}-11^x\) will be very close to \(11^{11x}\) itself.


Hello Bunuel,
Nice explanations. I am a little confused with the fact which you mentioned as "negligible" in this math. Wouldn't this deduction change the result of the answer?

Thanks. :)


The question asks about the closest value of the fraction among the options, not the exact value, so we can approximate.

Similar questions to practice:
the-value-of-10-8-10-2-10-7-10-3-is-closest-to-which-of-95082.html
which-of-the-following-is-closest-to-10180-1030-a-110224.html
which-of-the-following-best-approximates-the-value-of-q-if-99674.html
new-tough-and-tricky-exponents-and-roots-questions-125956-40.html
if-x-10-10-x-2-2x-7-3x-2-10x-2-is-closest-to-143897.html
1-0-0001-0-04-10-the-value-of-the-expression-above-is-59398.html
which-of-the-following-is-closest-in-value-to-64425.html
10-180-10-30-which-of-the-following-best-approximates-84309.html
if-x-2-b-8-8-8-6-for-which-of-the-following-b-values-160197.html
if-x-3-000-then-the-value-of-x-2x-1-is-closest-to-166128.html

Hope it helps.
Intern
Intern
Joined: 02 Dec 2017
Posts: 13
Own Kudos [?]: 1 [0]
Given Kudos: 45
Send PM
Re: NEW!!! Tough and tricky exponents and roots questions [#permalink]
Bunuel wrote:
3. If \(5^{10x}=4,900\) and \(2^{\sqrt{y}}=25\) what is the value of \(\frac{(5^{(x-1)})^5}{4^{-\sqrt{y}}}\)?
A. 14/5
B. 5
C. 28/5
D. 13
E. 14

First thing one should notice here is that \(x\) and \(y\) must be some irrational numbers (4,900 has other primes then 5 in its prime factorization and 25 doesn't have 2 as a prime at all), so we should manipulate with given expressions rather than to solve for x and y.

\(5^{10x}=4,900\) --> \((5^{5x})^2=70^2\) --> \(5^{5x}=70\)

\(\frac{(5^{(x-1)})^5}{4^{-\sqrt{y}}}=5^{(5x-5)}*4^{\sqrt{y}}=5^{5x}*5^{-5}*(2^{\sqrt{y}})^2=70*5^{-5}*25^2=70*5^{-5}*5^4=70*5^{-1}=\frac{70}{5}=14\)

Answer: E.


Hey there, just one remark. Why could 4sqrt/y not become by 2 x 2 sqrt/y? why is it definitely (2 sqrt/y)^2 ?

thanks
GMAT Club Bot
Re: NEW!!! Tough and tricky exponents and roots questions [#permalink]
   1   2   3   4   
Moderators:
Math Expert
92918 posts
Senior Moderator - Masters Forum
3137 posts

Powered by phpBB © phpBB Group | Emoji artwork provided by EmojiOne